2014 dxdy logo

Научный форум dxdy

Математика, Физика, Computer Science, Machine Learning, LaTeX, Механика и Техника, Химия,
Биология и Медицина, Экономика и Финансовая Математика, Гуманитарные науки


Правила форума


В этом разделе нельзя создавать новые темы.

Если Вы хотите задать новый вопрос, то не дописывайте его в существующую тему, а создайте новую в корневом разделе "Помогите решить/разобраться (М)".

Если Вы зададите новый вопрос в существующей теме, то в случае нарушения оформления или других правил форума Ваше сообщение и все ответы на него могут быть удалены без предупреждения.

Не ищите на этом форуме халяву, правила запрещают участникам публиковать готовые решения стандартных учебных задач. Автор вопроса обязан привести свои попытки решения и указать конкретные затруднения.

Обязательно просмотрите тему Правила данного раздела, иначе Ваша тема может быть удалена или перемещена в Карантин, а Вы так и не узнаете, почему.



Начать новую тему Ответить на тему На страницу 1, 2  След.
 
 предел последовательности
Сообщение25.10.2008, 11:11 
Аватара пользователя


01/12/07
172
Как посчитать предел:
$$\lim\limits_{n\to\infty}\max\limits_{x\in\{[0,{1/3]}}{|\sin(2\pi x^n)|}}}$$

 Профиль  
                  
 
 
Сообщение25.10.2008, 11:20 
Заслуженный участник
Аватара пользователя


11/04/08
2739
Физтех
Если я правильно понял условие, то при достаточно больших n \[
\left| {\sin \left( {2\pi x^n } \right)} \right| = \sin \left( {2\pi x^n } \right)
\] при \[
{x \in \left[ {0;\frac{1}
{3}} \right]}
\]. Тогда \[
\mathop {\max }\limits_{x \in \left[ {0;\frac{1}
{3}} \right]} \left| {\sin \left( {2\pi x^n } \right)} \right| \to 2\pi x^n ,n \to  + \infty 
\].

 Профиль  
                  
 
 
Сообщение25.10.2008, 11:22 
Аватара пользователя


01/12/07
172
Значит предел будет равен 0 при$x\in\[0,1/3]$?

 Профиль  
                  
 
 
Сообщение25.10.2008, 11:23 
Заслуженный участник
Аватара пользователя


11/04/08
2739
Физтех
Да.

 Профиль  
                  
 
 
Сообщение25.10.2008, 11:25 
Аватара пользователя


01/12/07
172
А если $x\in\{[0,1]}$?

 Профиль  
                  
 
 
Сообщение25.10.2008, 11:30 
Заслуженный участник
Аватара пользователя


11/04/08
2739
Физтех
При \[
x \in \left[ {0;1} \right)
\] все то же самое. А что будет при x=1?

 Профиль  
                  
 
 
Сообщение25.10.2008, 11:37 
Заслуженный участник
Аватара пользователя


30/10/07
1221
Самара/Москва
matan писал(а):
А если $x\in\{[0,1]}$?

Для любого $n$ выбираем $x=(1/4)^{1/n}\in[0,1)\subset[0,1]$.. дальше сами

ShMaxG, не то же самое. Случаи полуинтервала и отрезка как раз одинаковы.

Добавлено спустя 5 минут 47 секунд:

ShMaxG писал(а):
Тогда \[
\mathop {\max }\limits_{x \in \left[ {0;\frac{1}
{3}} \right]} \left| {\sin \left( {2\pi x^n } \right)} \right| \to 2\pi x^n ,n \to  + \infty 
\].

Это как это так? левая часть чиcловая последовательность, а правая - функция от $x$ ?? Даже не функция, а функц. посл.-ть ?!

 Профиль  
                  
 
 
Сообщение25.10.2008, 11:43 
Заслуженный участник
Аватара пользователя


11/04/08
2739
Физтех
2Henrylee

Да, вы правы, я ошибся.:oops:

 Профиль  
                  
 
 
Сообщение26.10.2008, 16:55 
Аватара пользователя


16/02/06
222
Украина
Давайте посчитаем $\max\limits_{x\in[0,\frac 1 3]}|sin(2\pi x^n)|$. Для этого рассмотрим функцию $f(x)=|sin(2\pi x^n)|=sin(2\pi x^n)$ - это не сложно показать, когда $x\in[0,\frac 1 3]$. Найдем $f'(x)$ и приравняем к $0$. Получим точки $0$ и $x^n=\frac k 2+\frac 1 4$. Нам не подходит $0$, проанализируем при скольких $k$ наши точки попадают в заданный отрезок. Выходит только при $k=0$.Значит наши точки экстремума есть $x^n=\frac 1 4$ и $\max\limits_{x\in[0,\frac 1 3]}|sin(2\pi x^n)|=sin \frac {\pi} 2=1.$

 Профиль  
                  
 
 
Сообщение26.10.2008, 17:05 
Заслуженный участник
Аватара пользователя


01/03/06
13626
Москва
citadeldimon в сообщении #153440 писал(а):
Значит наши точки экстремума есть $x^n=\frac 1 4$

А почему все эти точки лежат на отрезке $x\in[0,\frac 1 3]$?

 Профиль  
                  
 
 
Сообщение26.10.2008, 17:14 
Аватара пользователя


16/02/06
222
Украина
А разве не так? Надо просто решить два неравенства:
$\frac k 2 +\frac  1 4\geq 0$ и $\frac k 2 +\frac  1 4\leq (\frac 1 3)^n$. И не забыть что $k$ - целое.

 Профиль  
                  
 
 
Сообщение26.10.2008, 17:20 
Заслуженный участник
Аватара пользователя


01/03/06
13626
Москва
citadeldimon в сообщении #153448 писал(а):
А разве не так?
Не так:\[
x = \frac{1}{{\sqrt[n]{4}}} \to 1\;,\;n \to \infty 
\]

 Профиль  
                  
 
 
Сообщение26.10.2008, 17:33 
Аватара пользователя


16/02/06
222
Украина
Brukvalub вы правы, не досмотрел, провел грубую оценку, но что тогда выходит? Функция имеет максимум в крайней точке $x=\frac 1 3.$ И тогда $2\pi x^n\to 0, lim=0$Так?

 Профиль  
                  
 
 
Сообщение26.10.2008, 17:33 
Заслуженный участник
Аватара пользователя


01/03/06
13626
Москва
citadeldimon в сообщении #153457 писал(а):
Функция имеет максимум в крайней точке $x=\frac 1 3.$ Так?
Да.

 Профиль  
                  
 
 
Сообщение26.10.2008, 17:36 
Аватара пользователя


16/02/06
222
Украина
Спасибо Brukvalub за исправление ошибки, ответ граница равна нулю, но если $x\in[0,1]$, то ответ $1$.

 Профиль  
                  
Показать сообщения за:  Поле сортировки  
Начать новую тему Ответить на тему  [ Сообщений: 16 ]  На страницу 1, 2  След.

Модераторы: Модераторы Математики, Супермодераторы



Кто сейчас на конференции

Сейчас этот форум просматривают: нет зарегистрированных пользователей


Вы не можете начинать темы
Вы не можете отвечать на сообщения
Вы не можете редактировать свои сообщения
Вы не можете удалять свои сообщения
Вы не можете добавлять вложения

Найти:
Powered by phpBB © 2000, 2002, 2005, 2007 phpBB Group